Đến nội dung

quanguefa nội dung

Có 565 mục bởi quanguefa (Tìm giới hạn từ 09-06-2020)



Sắp theo                Sắp xếp  

#628537 $M=(x-1)^2+(y-1)^2+\frac{2016+2xy\sqrt{x+y+1}...

Đã gửi bởi quanguefa on 20-04-2016 - 19:07 trong Bất đẳng thức và cực trị

Bài1:

Cho $x,y\in \mathbb{R}$ thoả mãn $x+y=26\sqrt{x-3}+3\sqrt{y-2013}+2016$. Tìm min, max:

$M=(x-1)^2+(y-1)^2+\frac{2016+2xy\sqrt{x+y+1}}{\sqrt{x+y+1}}$

 

Bài 2:

Cho $a,b,c>0$ thỏa mãn $\frac{4a}{b}(1+\frac{2c}{b})+\frac{b}{a}(1+\frac{c}{a})=6$. Tìm min:

$P=\frac{bc}{a(b+2c)}+\frac{2ca}{b(c+a)}+\frac{2ab}{c(2a+b)}$

Bài 2

https://www.facebook...95&l=888aa5ed78

https://www.facebook...84&l=fec9f2c0a4




#628409 Tổng hợp các bài BĐT trong các đề thi thử THPT Quốc Gia môn Toán năm 2017

Đã gửi bởi quanguefa on 20-04-2016 - 01:10 trong Bất đẳng thức và cực trị

Bài 34:

Cho x, y, z thực dương: $x+y+1=z$

Tìm GTNN của: $P=\sum \frac{x^3}{x+yz}+\frac{14}{(z+1)\sqrt{(x+1)(y+1)}}$

Đây là cách của mình, về bản chất cũng giống cách của bạn Hùng nhưng biến đổi khác 1 tý

Thay z=x+y+1 vào biểu thức ta có:

 

$\sum \frac{x^3}{x+yz}=\frac{x^3}{(x+y)(y+1)}+\frac{y^3}{(x+y)(x+1)}+\frac{z^3}{(x+1)(y+1)}$

 

                     $=\frac{x^3(x+1)+y^3(y+1)+z^3(x+y)}{(x+y)(x+1)(y+1)}$

 

                     $=\frac{x^3+y^3+x^4+y^4+z^3(x+y)}{(x+y)(x+y+xy+1)}$

 

                     $\geq \frac{\frac{(x+y)^4}{8}+\frac{(x+y)^3}{4}+z^3(x+y)}{(x+y)[\frac{(x+y)^2}{4}+x+y+1]}$

 

                     $=\frac{(x+y)^3+2(x+y)^2+z^3}{2(x+y)^2+8x+8y+8}=\frac{(z-1)^2(z+1)+z^3}{2(z+1)^2}$

 

Phần còn lại làm giống bạn hùng




#628328 Tổng hợp các bài BĐT trong các đề thi thử THPT Quốc Gia môn Toán năm 2017

Đã gửi bởi quanguefa on 19-04-2016 - 21:35 trong Bất đẳng thức và cực trị

Bài 37. (trích đề KSCL Quảng Nam)

Cho a, b, c dương thỏa: $\frac{4a}{b}(1+\frac{2c}{b})+\frac{b}{a}(1+\frac{c}{a})=6$

Tìm GTNN: $P=\frac{bc}{a(b+2c)}+\frac{2ca}{b(c+a)}+\frac{2ab}{c(2a+b)}$




#628316 Tổng hợp các bài BĐT trong các đề thi thử THPT Quốc Gia môn Toán năm 2017

Đã gửi bởi quanguefa on 19-04-2016 - 21:25 trong Bất đẳng thức và cực trị

Theo Cauchy_Swatch ta có : $\frac{x^3}{x+yz}+\frac{y^3}{y+xz}\geq \frac{(x+y)^3}{2(x+y+yz+xz)}=\frac{(x+y)^3}{2(x+y)(1+z)}=\frac{(x+y)^2}{2(z+1)}=\frac{(z-1)^2}{2(z+1)}$  (Do $x+y=z-1$)

 

 Mà $\frac{z^3}{z+xy}=\frac{4z^3}{4z+4xy}\geq \frac{4z^3}{4z+(x+y)^2}=\frac{4z^3}{4z+(z-1)^2}=\frac{4z^3}{(z+1)^2}$

 

  $(z+1)\sqrt{(x+1)(y+1)}\leq (z+1).\frac{x+y+2}{2}=\frac{(z+1)(z-1+2)}{2}=\frac{(z+1)^2}{2}= > \frac{14}{(z+1)\sqrt{(x+1)(y+1)}}\geq \frac{28}{(z+1)^2}$

 

Từ đó $= > P\geq \frac{(z-1)^2}{2(z+1)}+\frac{4z^3}{(z+1)^2}+\frac{28}{(z+1)^2}=> 2P\geq \frac{(z-1)^2}{z+1}+\frac{8z^3}{(z+1)^2}+\frac{56}{(z+1)^2}=\frac{(z-1)^2(z+1)+8z^3+56}{(z+1)^2}=f(z)$

 

  Ta chứng minh $f(z)\geq \frac{53}{4}< = > \frac{(z-1)^2(z+1)+8z^3+56}{(z+1)^2}\geq \frac{53}{4}$
$< = > \frac{9z^3-z^2-z+57}{z^2+2z+1}\geq \frac{53}{4}< = > 36z^3-4z^2-4z+228\geq 53z^2+106z+53$
$< = > 36z^3-57z^2-110z+175\geq 0< = > 12z^2(3z-5)+z(3z-5)-35(3z-5)\geq 0$
$< = > (3z-5)(12z^2+z-35)\geq 0< = > (3z-5)(4z(3z-5)+7(3z-5))\geq 0$
$< = > (3z-5)^2(4z+7)\geq 0$ (Luôn đúng)

 

 Do đó $2P\geq f(z)\geq \frac{53}{4}= > P\geq \frac{53}{8}= > P_{Min}=\frac{53}{8}< = > \left\{\begin{matrix} x=y & & \\ z=\frac{5}{3} & & \\ x+y+1=z & & \end{matrix}\right.< = > \left\{\begin{matrix} x=y=\frac{1}{3} & \\ z=\frac{5}{3} & \end{matrix}\right.$

Sr m.n mình ghi đề bị nhầm nhiều quá, để sửa lại ngay, ban hùng tự sửa đề làm luôn rồi mà lại ko nói :3

hùng ơi chỗ này hơi khó hiểu, giải thích giùm $\frac{x^3}{x+yz}+\frac{y^3}{y+xz}\geq \frac{(x+y)^3}{2(x+y+yz+xz)}$




#628302 Tổng hợp các bài BĐT trong các đề thi thử THPT Quốc Gia môn Toán năm 2017

Đã gửi bởi quanguefa on 19-04-2016 - 21:06 trong Bất đẳng thức và cực trị

phương pháp chuẩn hóa bất đẳng thức là gì vậy ai giải thích được ko

bạn xem ở đây http://diendantoanho...-chuẩn-hoa-bdt/

và đây: http://diendantoanho...-bất-đẳng-thức/

Nhìu tài liệu về BĐT cũng có nói tới cái này mà bạn

 

klq nhưng câu 35 chả mang dáng dấp đề đại học tý nào :3




#627988 Tổng hợp các bài BĐT trong các đề thi thử THPT Quốc Gia môn Toán năm 2017

Đã gửi bởi quanguefa on 18-04-2016 - 19:08 trong Bất đẳng thức và cực trị

Mọi người cho mình hỏi ngoài lề (không biết hỏi chỗ nào cho hợp lý, mod thông cảm)

1.Thi Đại Học được dùng pp chuẩn hóa BĐT không (mặc dù cũng hiếm khi cần đến).

2.Thi ĐH mình trình bày ngắn gọi như kiểu trình bày trong barem điểm thì có full điểm không (ý mình là trình bày tắt (đủ ý) kiểu đáp án chứ không phải là làm theo cách giống đáp án). Tại mình thấy đáp án thường làm gắn gọn vắn tắt (đôi khi bỏ bước nhỏ), làm kiểu đó mà kiểm tra là thầy mình trừ điểm liền :)




#627925 Tổng hợp các bài BĐT trong các đề thi thử THPT Quốc Gia môn Toán năm 2017

Đã gửi bởi quanguefa on 18-04-2016 - 13:06 trong Bất đẳng thức và cực trị

Bài 34:

Cho x, y, z thực dương: $x+y+1=z$

Tìm GTNN của: $P=\sum \frac{x^3}{x+yz}+\frac{14}{(z+1)\sqrt{(x+1)(y+1)}}$




#627902 Tìm GTNN của $P=\frac{a^{2}+b^{2}+c^{...

Đã gửi bởi quanguefa on 18-04-2016 - 09:24 trong Bất đẳng thức và cực trị

hướng dẫn mình giải bất phương trình này đi, cái điều kiện mình ghi nhầm đó, không phải là $a+b+c=3$ đâu, phải là $3\left( {{a}^{2}}+{{b}^{2}}+{{c}^{2}} \right)+ab+bc+ca=12$ mới đúng

Bài này mình nghĩ chỉ cần dồn biến là ra cả MIN, MAX

Đặt $t=ab+bc+ca$. Dễ dàng chứng minh $0\leq t\leq 3$

Ta có: $\sum a^2=\frac{12-t}{3}$,

           $\sum a=\sqrt{\sum a^2+2\sum ab}=\sqrt{\frac{12-t}{3}+2t}$

Thay vào biểu thức vào rút gọn thu được

           $P=f(t)=\frac{12-t}{\sqrt{15t+36}}+t$

Dễ dàng c/m: $f't)>0$ với t thuộc [0;2]

Từ đó suy ra min, max




#627896 Tìm GTNN của: $P=\sum \frac{1}{1+a^2}$

Đã gửi bởi quanguefa on 18-04-2016 - 00:34 trong Bất đẳng thức và cực trị

Cho a, b, c dương; ab+bc+ca=3. Tìm GTNN của: $P=\sum \frac{1}{1+a^2}$




#627892 $P=\frac{16}{\sqrt{x^{2}y^{...

Đã gửi bởi quanguefa on 18-04-2016 - 00:27 trong Bất đẳng thức và cực trị

Cho $x,y,z$ là các số thực không âm thỏa mãn $x^{2}+y^{2}+z^{2}=3$. Tìm GTNN của 

   $P=\frac{16}{\sqrt{x^{2}y^{2}+y^{2}z^{2}+z^{2}x^{2}+1}}+\frac{xy+yz+zx+1}{x+y+z}$

đã có tại đây: http://diendantoanho...e-3#entry627891




#627891 Tổng hợp các bài BĐT trong các đề thi thử THPT Quốc Gia môn Toán năm 2017

Đã gửi bởi quanguefa on 18-04-2016 - 00:21 trong Bất đẳng thức và cực trị

Bài 24:Cho $x,y,z$ là các số thực không âm thỏa mãn $x^{2}+y^{2}+z^{2}=3$. Tìm GTNN của 

   $P=\frac{16}{\sqrt{x^{2}y^{2}+y^{2}z^{2}+z^{2}x^{2}+1}}+\frac{xy+yz+zx+1}{x+y+z}$

 

_Đề thi thử Chu Văn An Sơn La_

Bài 24:

Đầu tiên ta chứng minh: $\sum x\geq \sum x^2y^2$

Ta có: $\sum x^2y^2=\frac{(\sum x^2)^{2}-\sum x^{4}}{2}=\frac{9-\sum x^{4}}{2}$

Cần chứng minh: $\frac{9-\sum x^{4}}{2}\leq\sum  x\Leftrightarrow 2\sum x+\sum x^4\geq 9$

BĐT này đúng nhờ đánh giá AM-GM: $x^4+x+x\geq 3x^2$

Từ đó ta có: $P\geq \frac{16}{\sqrt{\sum x+1}}+\frac{\frac{(\sum x)^2-\sum x^2}{2}+1}{\sum x}=\frac{16}{\sqrt{\sum x+1}}+\frac{(\sum x)^2-1}{2\sum x}$

Đặt $t=x+y+z$ $(t\leq 3)$. Khi đó: $P\geq f(t)=\frac{16}{\sqrt{t+1}}+\frac{t^2-1}{2t}$

Tính đạo hàm suy ra f(t) nghịch biến, suy ra: $f(t)_{min}=f(3)=\frac{28}{3}\Rightarrow P\geq \frac{28}{3}$

Vậy.... dấu bằng xảy ra khi x=y=z=1

 

Sử dụng hơi nhiều dấu sigma các bạn thông cảm




#627887 Tổng hợp các bài BĐT trong các đề thi thử THPT Quốc Gia môn Toán năm 2017

Đã gửi bởi quanguefa on 17-04-2016 - 23:40 trong Bất đẳng thức và cực trị

Bài 33: (lớp toán Lý Thái Tổ, thi thử lần thứ 14 năm 2015).

Cho x, y, z thực dương thỏa mãn $x^{2}+y^{2}+z^{2}=2$. Tìm GTNN của biểu thức

$P=\frac{3x^2+3y^2}{8}+\frac{2z}{x+y}-\frac{z^2+z}{(x+1)(y+1)}-\sqrt{3}z$




#627885 Tổng hợp các bài BĐT trong các đề thi thử THPT Quốc Gia môn Toán năm 2017

Đã gửi bởi quanguefa on 17-04-2016 - 23:31 trong Bất đẳng thức và cực trị

Bài 31: (THPT Hàn Thuyên lần 1)

Cho các số thực $x,y,z$ thỏa $x>2$, $y>1$, $z>0$

Tìm giá trị lớn nhất của:

 

$P= \frac{1}{\sqrt{x^{2}+y^{2}+z^{2}-2(2x+y-3)}}-\frac{1}{y(x-1)(z+1)}$

Bài 31: Dạng này xuất hiện trong nhiều đề thi thử rồi, trong tuyển tập bộ 3 câu khó của VMF cũng xuất hiện

Đề bị nhầm 1 chút nha, sửa lại: $P= \frac{1}{2\sqrt{x^{2}+y^{2}+z^{2}-2(2x+y-3)}}-\frac{1}{y(x-1)(z+1)}$
 
Đặt $a=x-2$, $b=y-1$, $c=z$. Ta có a, b, c>0
$P=\frac{1}{2\sqrt{\sum a^{2}+1}}-\frac{1}{(a+1)(b+1)(c+1)}=\frac{1}{\sqrt{(1+1+1+1)(a^{2}+b^{2}+c^{2}+1)}}-\frac{1}{(a+1)(b+1)(c+1)}\leq \frac{1}{a+b+c+1}-\frac{27}{(a+1+b+1+c+1)^{3}}=\frac{1}{a+b+c+1}-\frac{27}{(a+b+c+3)^{3}}$
Đăt $t=a+b+c$ $(t>0)$, khi đó: $P=f(t)=\frac{1}{t+1}-\frac{27}{(t+3)^{3}}$
Có: $f'(t)=0\Leftrightarrow t=3\Rightarrow f(t)_{max}=f(3)=\frac{1}{8}$
Vậy $P\leq \frac{1}{8}$. Đẳng thức xảy ra khi x=3, y=2, z=1
 
P/s: sao mình thấy nhiều câu thi thử trông nó có vẻ đậm chất thi HSG quá vậy nhỉ @@ rồi một số lời giải các bạn thì ôi thôi rồi luôn, holder, schur chóng hết cả mặt...



#627863 $\frac{x^3y^3}{(x+yz)(y+xz)(z+xy)^2}$

Đã gửi bởi quanguefa on 17-04-2016 - 22:08 trong Bất đẳng thức và cực trị

Mình học dốt lắm nên mấy cái giới hạn gì gì đó mình chưa biết :V :v , 

Èo bạn mới 2k à, từ từ hãy làm mấy cái này chứ

Thì bạn cứ thử thay x, y lớn vào xem, P sẽ dần về 0 (nhỏ dần), mà P luôn lớn hơn 0. Như vậy không tồn tại MIN




#627859 $\frac{x^3y^3}{(x+yz)(y+xz)(z+xy)^2}$

Đã gửi bởi quanguefa on 17-04-2016 - 21:57 trong Bất đẳng thức và cực trị

ảnh đề là tìm min đó bạn 

Vậy có thể làm như thế này không, ta có P>0 mà khi x=y và tiến về vô cực thì limP=0

Như vậy không tồn tại MIN

 

Đấy chỉ là đề thi thử của trường, nhầm là chuyện bình thường bạn à. Câu đó cũng là câu thi thử của trường mình (mới thi hôm trước). Trường mình cũng ghi sai đề nhưng sai chỗ z+xy (đề đúng là bình phương lên), nhưng yêu cầu đề bài là tìm max.




#627858 $MinP=\sqrt{(x-1)(y-1)}+\frac{18xy}{x...

Đã gửi bởi quanguefa on 17-04-2016 - 21:54 trong Bất đẳng thức và cực trị

Cho $x\geq 1,y\geq 1,z>0$ thỏa mãn $x+y+xyz=xy$

Tìm $MinP=\sqrt{(x-1)(y-1)}+\frac{18xy}{x+y+2xyz}$

Ta có: $z=1-\frac{1}{x}-\frac{1}{y}\Rightarrow \frac{1}{x}+\frac{1}{y}=1-z< 1$

$P=\sqrt{(x-1)(y-1)}+\frac{18xy}{x+y+2xyz}=\sqrt{xy-(x+y)+1}+\frac{18xy}{2xy-(x+y)}=\sqrt{xy}.\sqrt{1-(\frac{1}{x}+\frac{1}{y})+\frac{1}{xy}}+\frac{18}{2-(\frac{1}{x}+\frac{1}{y})}$

Đặt $a=\frac{1}{x};b=\frac{1}{y}$ ($a,b\leq 1$;  $a+b< 1$)

Khi đó: $P=\frac{18}{2-(a+b)}+\frac{1}{\sqrt{ab}}\sqrt{1-(a+b)+ab}=\frac{18}{2-(a+b)}+\sqrt{\frac{1-(a+b)}{ab}+1}\geq \frac{18}{2-(a+b)}+\sqrt{\frac{4[1-(a+b)]}{(a+b)^{2}}+1}$

Đặt t=a+b, ta có t<1. Và: $P=f(t)=\sqrt{\frac{4(1-t)}{t^2}+1}+\frac{18}{2-t}$

Tính đạo hàm và lập BBT thu được: $f(t)_{min}=f(\frac{1}{2})=15\Rightarrow P\geq 15$

Đẳng thức xảy ra khi: x=y=4, z=0,5




#627812 $\frac{x^3y^3}{(x+yz)(y+xz)(z+xy)^2}$

Đã gửi bởi quanguefa on 17-04-2016 - 20:29 trong Bất đẳng thức và cực trị

Cho 3 số thực dương x,y,z thỏa mãn : $x+y-z=-1$
Tìm Min: $\frac{x^3y^3}{(x+yz)(y+xz)(z+xy)^2}$

Mình nghĩ bạn ghi đề sai rồi, tìm MAX mới đúng :)

Đặt biểu thức là P, thay z=x+y+1 vào biểu thức rồi phân tích nhân tử ta được:

$P=\frac{x^3.y^3}{(x+y)^{2}(x+1)^{3}(y+1)^{3}}=\frac{x^3.y^3}{(x+y)^{2}(xy+x+y+1)^{3}}\leq  \frac{x^3.y^3}{4xy.(xy+2\sqrt{xy}+1)^{3}}=\frac{x^2y^2}{4(\sqrt{xy}+1)^{6}}=[\frac{xy}{2(\sqrt{xy}+1)^{3}}]^{2}$

Đặt $t=\sqrt{xy}$. Xét hàm $f(t)=\frac{t^2}{(t+1)^3}$ với t dương

Tính đạo hàm lập BBT ta thu được: $f(t)_{max}=f(2)\doteq \frac{4}{27}$

Suy ra: $P\leq \frac{4}{729}$

Đẳng thức xảy ra khi x=y=2, z=5

 

Còn min thì cũng từ việc thay z=x+y+1 ta thấy mẫu bậc lớn hơn tử nên min sẽ là 0 khi x, y tiến về vô cực




#627793 $4(3x+\sqrt{9x^{2}+4})=\frac{1}...

Đã gửi bởi quanguefa on 17-04-2016 - 19:38 trong Phương trình - hệ phương trình - bất phương trình

Giải PT: $4(3x+\sqrt{9x^{2}+4})=\frac{1}{x}+\frac{9x}{x^{2}+1}$




#627768 Cho tam giác ABC nội tiếp đường tròn tâm K...Tìm tọa độ A

Đã gửi bởi quanguefa on 17-04-2016 - 17:42 trong Phương pháp tọa độ trong mặt phẳng

Cho tam giác ABC nội tiếp đường tròn tâm K. Hai tiếp tuyến tại B và C cắt nhau tại M. N là giao của AM và BC. Biết B(8;7); C(9;2); N thuộc đường thẳng $43x-43y-193=0$; A thuộc đường thẳng $x+y-5=0$. Tìm tọa độ A




#627637 $\frac{8(a+b+c)^{2}}{a^2+b^2+c^2}+...

Đã gửi bởi quanguefa on 17-04-2016 - 00:21 trong Bất đẳng thức và cực trị

Bằng cách phân tích thành các tổng bình phương ta cần chứng minh:
$\sum (b-c)^2(\frac{3}{bc}-\frac{8}{a^2+b^2+c^2})\geq 0$
Ta có:
$S_a=3(a^2+b^2+c^2)-8bc$
$S_b=3(a^2+b^2+c^2)-8ca$
$S_c=3(a^2+b^2+c^2)-8ab$
$Th1:S_b=3(a^2+b^2+c^2)-8ca\geq 0$ , ta cần chứng minh:
$S_a+S_b=6\sum a^2-8c(a+b)=6(c-\frac{2}{3}(a+b))^2+6(a^2+b^2)-\frac{8}{3}(a+b)^2\geq 0$
$S_b+S_c=6\sum a^2-8a(b+c)=6(a-\frac{2}{3}(b+c))^2+6(b^2+c^2)-\frac{8}{3}(b+c)^2\geq 0$
$Th2: S_b=3(a^2+b^2+c^2)-8ca< 0$, ta cần chứng minh:
$S_a+2S_b> 0$
$Sc+2S_b\geq 0$
Công việc này khá đơn giản bạn chỉ cần xét delta <=0 cho pt bậc 2 
Đẳng thức xảy ra khi a=b=c
 

đây là 1 câu từ đề thi thử DH, có lẽ không phải dùng đến những kiến thức khủng như vậy @@

Thật tình là mình ko đủ trình đọc hết lời giải của bạn, dù gì cũng cảm ơn bạn

Mong nhận được một lời giải sơ cấp từ các cao thủ :)




#627388 $\frac{8(a+b+c)^{2}}{a^2+b^2+c^2}+...

Đã gửi bởi quanguefa on 16-04-2016 - 06:36 trong Bất đẳng thức và cực trị

Cho a, b, c dương. CM: $\frac{8(a+b+c)^{2}}{a^2+b^2+c^2}+\frac{3(a+b)(b+c)(c+a)}{abc}\geq 48$




#626246 $\sum \frac{1}{2x+y+3}\leq \frac...

Đã gửi bởi quanguefa on 10-04-2016 - 08:26 trong Bất đẳng thức và cực trị

 Giải tương tự như ở đây nhé , giống phần đầu, phần sau hơi khác tí

http://diendantoanho...2a23leq-frac12/

Bạn có thể giải cụ thể cho mình được không

À thôi ok rồi bạn




#626219 $\sum \frac{1}{2x+y+3}\leq \frac...

Đã gửi bởi quanguefa on 09-04-2016 - 23:03 trong Bất đẳng thức và cực trị

Cho x,y,z dương thỏa xyz=1

CMR: $\sum \frac{1}{2x+y+3}\leq \frac{1}{2}$




#626194 Tìm Min, Max $P=\sqrt{(a-1)(b-5)}$

Đã gửi bởi quanguefa on 09-04-2016 - 21:27 trong Bất đẳng thức và cực trị

Cho 2 số thực a,b thỏa mãn $a+b+25=8(\sqrt{a-1}+\sqrt{b-5})$

Tìm Min, Max của $P=\sqrt{(a-1)(b-5)}$

Câu 5 đề hsg tỉnh quảng ngãi

Mình đã giải chi tiết min, max tại đây: http://diendantoanho...16/#entry626130




#626191 Đề thi HSG lớp 11 tỉnh Quảng Ngãi 2015-2016

Đã gửi bởi quanguefa on 09-04-2016 - 21:23 trong Thi HSG cấp Tỉnh, Thành phố. Olympic 30-4. Đề thi và kiểm tra đội tuyển các cấp.

 

SỞ GIÁO DỤC VÀ ĐÀO TẠO QUẢNG NGÃI

KỲ THI CHỌN HỌC SINH GIỎI LỚP 11 CẤP TỈNH NĂM HỌC 2015-2016

MÔN THI: TOÁN

______________   

Thời gian: 180 phút 

Ngày thi: 06-04-2016

 

 

 

Bài 5 (3,0 điểm). Cho 2 số thực x, y thỏa mãn điều kiện $x+y+25=8(\sqrt{x-1}+\sqrt{y-5})$

Tìm GTLN và GTNN của biểu thức: $P=\sqrt{(x-1)(y-5)}$

 

 

Em xin chém câu 5.

 

Đầu tiên đặt:$a=\sqrt{x-1}, b=\sqrt{y-5}$

Suy ra: $a^{2}+b^{2}+31=8(a+b)$    $(*)$ và $P=ab$

 

Đến đây ta có dạng khá quen thuộc và thật sự nếu có máy tính thì khỏe @@

Tư tưởng chung cho dạng này là đặt S, P. Thay vì thế nếu quen tay ta dồn biến cho tiện. 

Đặt: $t=a^{2}+b^{2}$. Ta có: 

$a+b\leq \sqrt{2(a^{2}+b^{2})}\Leftrightarrow \frac{t+31}{8}\leq \sqrt{2t}\Leftrightarrow 33-8\sqrt{2}\leq t\leq 33+8\sqrt{2}$

Mà: $P=\frac{(a+b)^{2}-(a^{2}+b^{2})}{2}=\frac{(\frac{t+31}{8})^{2}-t}{2}=\frac{t^{2}-2t+961}{128}$

Xét hàm: $f(t)=t^{2}-2t$ với $33-8\sqrt{2}\leq t\leq 33+8\sqrt{2}$

Hàm đồng biến trên đoạn đang xét, suy ra: $f(33-8\sqrt{2})\leq f(t)\leq f(33+8\sqrt{2})$ (chỗ này có thể lập bảng biến thiên của tam thức bậc 2 cũng được)

Từ đó suy ra: $\frac{33-8\sqrt{2}}{2}\leq P\leq \frac{33+8\sqrt{2}}{2}$

Đến đây tìm điểm rơi dựa vào điểm rơi của t và $(*)$

 

P/S: h mới thấy đặt $t=\sqrt{a^{2}+b^{2}}$ thì tính toán sẽ đơn giản hơn.